Bạn chưa đăng nhập. Vui lòng đăng nhập để hỏi bài

Những câu hỏi liên quan
lu nguyễn
Xem chi tiết
Nguyễn Việt Lâm
3 tháng 4 2020 lúc 20:21

\(a=\lim\limits_{x\rightarrow-3}\frac{x^2+2x-3}{x\left(x+3\right)\left(x-\sqrt{3-2x}\right)}=\lim\limits_{x\rightarrow-3}\frac{\left(x-1\right)\left(x+3\right)}{x\left(x+3\right)\left(x-\sqrt{3-2x}\right)}=\lim\limits_{x\rightarrow-3}\frac{x-1}{x\left(x-\sqrt{3-2x}\right)}=-\frac{2}{9}\)

\(b=\lim\limits_{x\rightarrow0}\frac{\sqrt{x+9}-3+\sqrt{x+16}-4}{x}=\lim\limits_{x\rightarrow0}\frac{\frac{x}{\sqrt{x+9}+3}+\frac{x}{\sqrt{x+16}+4}}{x}=\lim\limits_{x\rightarrow0}\left(\frac{1}{\sqrt{x+9}+3}+\frac{1}{\sqrt{x+16}+4}\right)=\frac{7}{24}\)

\(c=\lim\limits_{x\rightarrow\frac{1}{2}}\frac{8x^2-1}{6x^2-5x+1}\) ko phải dạng vô định, đề bài là \(8x^2\) hay \(8x^3\) bạn?

\(d=\lim\limits_{x\rightarrow0}\frac{\left(\sqrt{x^2+1}-1\right)\left(\sqrt{x^2+1}+1\right)\left(4+\sqrt{x^2+16}\right)}{\left(4-\sqrt{x^2+16}\right)\left(4+\sqrt{x^2+16}\right)\left(\sqrt{x^2+1}+1\right)}\)

\(=\lim\limits_{x\rightarrow0}\frac{x^2\left(4+\sqrt{x^2+16}\right)}{-x^2\left(\sqrt{x^2+1}+1\right)}=\lim\limits_{x\rightarrow0}\frac{4+\sqrt{x^2+16}}{-\sqrt{x^2+1}-1}=\frac{8}{-2}=-4\)

Khách vãng lai đã xóa
Phụng Nguyễn Thị
Xem chi tiết
Akai Haruma
12 tháng 5 2020 lúc 22:25

Lời giải:

\(\lim\limits _{x\to +\infty}\sqrt{\frac{3x^4+4x^5+2}{9x^5+5x^4+4}}=\lim\limits _{x\to +\infty}\sqrt{\frac{\frac{3}{x}+4+\frac{2}{x^5}}{9+\frac{5}{x}+\frac{4}{x^5}}}=\sqrt{\frac{4}{9}}=\frac{2}{3}\)

Đáp án B.

Trần Phương Thảo
Xem chi tiết
Nguyễn Việt Lâm
15 tháng 3 2020 lúc 23:15

Bài 1:

\(a=\lim\limits_{x\rightarrow+\infty}\frac{\frac{1}{x}+\frac{2}{\sqrt{x}}-1}{1+\frac{3}{x}}=-1\)

\(b=\lim\limits_{x\rightarrow+\infty}\frac{1+\frac{3}{x^2}-\frac{1}{x^3}}{\frac{1}{\sqrt{x}}+\frac{1}{x^2}}=\frac{1}{0}=+\infty\)

\(c=\lim\limits_{x\rightarrow-\infty}\frac{1-2\sqrt{\frac{1}{x^2}-\frac{1}{x}}}{\frac{1}{x}-1}=\frac{1}{-1}=-1\)

Bài 2:

\(a=\lim\limits_{x\rightarrow0}\frac{1-cosx}{1-cos3x}=\lim\limits_{x\rightarrow0}\frac{sinx}{3sin3x}=\lim\limits_{x\rightarrow0}\frac{\frac{sinx}{x}}{9.\frac{sin3x}{3x}}=\frac{1}{9}\)

\(b=\lim\limits_{x\rightarrow0}\frac{cotx-sinx}{x^3}=\frac{\infty}{0}=+\infty\)

\(c=\lim\limits_{x\rightarrow\infty}\frac{sinx}{2x}\)

\(\left|sinx\right|\le1\Rightarrow\left|\frac{sinx}{2x}\right|\le\frac{1}{\left|2x\right|}\)

\(\lim\limits_{x\rightarrow\infty}\frac{1}{2\left|x\right|}=0\Rightarrow\lim\limits_{x\rightarrow\infty}\frac{sinx}{2x}=0\)

Khách vãng lai đã xóa
7 . 0 . 7
Xem chi tiết
Akai Haruma
25 tháng 1 2020 lúc 21:59

Lời giải:
a)

\(\lim\limits_{x\to-1}\frac{\sqrt[3]{x}+1}{2x^2+5x+3}=\lim\limits_{x\to-1}\frac{x+1}{\left(\sqrt[3]{x^2}-\sqrt[3]{x}+1\right)\left(x+1\right)\left(2x+3\right)}\)

\(\lim\limits_{x\to-1}\frac{1}{\left(\sqrt[3]{x^2}-\sqrt[3]{x}+1\right)\left(2x+3\right)}=\frac{1}{\left(\sqrt[3]{\left(-1\right)^2}-\sqrt[3]{-1}+1\right)\left(2.-1+3\right)}=\frac{1}{3}\)

b)

\(\lim\limits_{x\to1}\frac{\sqrt[3]{x^2}-2\sqrt[3]{x}+1}{\left(x-1\right)^2}=\lim\limits_{x\to1}\frac{\left(\sqrt[3]{x}-1\right)^2}{\left(x-1\right)^2}=\lim\limits_{x\to1}\frac{\left(x-1\right)^2}{\left(\sqrt[3]{x^2}+\sqrt[3]{x}+1\right)^2\left(x-1\right)^2}\)

\(=\lim\limits_{x\to1}\frac{1}{\left(\sqrt[3]{x^2}+\sqrt[3]{x}+1\right)^2}=\frac{1}{\left(1+1+1\right)^2}=\frac{1}{9}\)

c)

\(\lim_{x\to 1}\frac{\sqrt[4]{x}-1}{x^3+x^2-2}=\lim_{x\to 1}\frac{\sqrt[4]{x}-1}{(x-1)(x^2+2x+2)}=\lim_{x\to 1}\frac{x-1}{(\sqrt{x}+1)(\sqrt[4]{x}+1)(x-1)(x^2+2x+2)}\)

\(=\lim_{x\to 1}\frac{1}{(\sqrt{x}+1)(\sqrt[4]{x}+1)(x^2+2x+2)}=\frac{1}{(1+1)(1+1)(1+2.1+2)}=\frac{1}{20}\)

d)

\(\lim_{x\to -2}\frac{\sqrt[3]{2x+12}+x}{x^2+2x}=\lim_{x\to -2}\frac{2x+12+x^3}{(\sqrt[3]{(2x+12)^2}-x\sqrt[3]{2x+12}+x^2).x(x+2)}\)

\(=\lim_{x\to -2}\frac{(x+2)(x^2-2x+6)}{(\sqrt[3]{(2x+12)^2}-x\sqrt[3]{2x+12}+x^2).x(x+2)}=\lim_{x\to -2}\frac{x^2-2x+6}{(\sqrt[3]{(2x+12)^2}-x\sqrt[3]{2x+12}+x^2).x}\)

\(=\frac{-7}{12}\)

Khách vãng lai đã xóa
Akai Haruma
7 tháng 1 2020 lúc 16:38

Lời giải:
a)

\(\lim\limits_{x\to-1}\frac{\sqrt[3]{x}+1}{2x^2+5x+3}=\lim\limits_{x\to-1}\frac{x+1}{\left(\sqrt[3]{x^2}-\sqrt[3]{x}+1\right)\left(x+1\right)\left(2x+3\right)}\)

\(\lim\limits_{x\to-1}\frac{1}{\left(\sqrt[3]{x^2}-\sqrt[3]{x}+1\right)\left(2x+3\right)}=\frac{1}{\left(\sqrt[3]{\left(-1\right)^2}-\sqrt[3]{-1}+1\right)\left(2.-1+3\right)}=\frac{1}{3}\)

b)

\(\lim\limits_{x\to1}\frac{\sqrt[3]{x^2}-2\sqrt[3]{x}+1}{\left(x-1\right)^2}=\lim\limits_{x\to1}\frac{\left(\sqrt[3]{x}-1\right)^2}{\left(x-1\right)^2}=\lim\limits_{x\to1}\frac{\left(x-1\right)^2}{\left(\sqrt[3]{x^2}+\sqrt[3]{x}+1\right)^2\left(x-1\right)^2}\)

\(=\lim\limits_{x\to1}\frac{1}{\left(\sqrt[3]{x^2}+\sqrt[3]{x}+1\right)^2}=\frac{1}{\left(1+1+1\right)^2}=\frac{1}{9}\)

c)

\(\lim_{x\to 1}\frac{\sqrt[4]{x}-1}{x^3+x^2-2}=\lim_{x\to 1}\frac{\sqrt[4]{x}-1}{(x-1)(x^2+2x+2)}=\lim_{x\to 1}\frac{x-1}{(\sqrt{x}+1)(\sqrt[4]{x}+1)(x-1)(x^2+2x+2)}\)

\(=\lim_{x\to 1}\frac{1}{(\sqrt{x}+1)(\sqrt[4]{x}+1)(x^2+2x+2)}=\frac{1}{(1+1)(1+1)(1+2.1+2)}=\frac{1}{20}\)

d)

\(\lim_{x\to -2}\frac{\sqrt[3]{2x+12}+x}{x^2+2x}=\lim_{x\to -2}\frac{2x+12+x^3}{(\sqrt[3]{(2x+12)^2}-x\sqrt[3]{2x+12}+x^2).x(x+2)}\)

\(=\lim_{x\to -2}\frac{(x+2)(x^2-2x+6)}{(\sqrt[3]{(2x+12)^2}-x\sqrt[3]{2x+12}+x^2).x(x+2)}=\lim_{x\to -2}\frac{x^2-2x+6}{(\sqrt[3]{(2x+12)^2}-x\sqrt[3]{2x+12}+x^2).x}\)

\(=\frac{-7}{12}\)

Khách vãng lai đã xóa
Trần Phương Thảo
Xem chi tiết
Nguyễn Việt Lâm
15 tháng 3 2020 lúc 22:59

Bài 1:

\(a=\lim\limits_{x\rightarrow-\infty}\frac{2\left|x\right|+1}{3x-1}=\lim\limits_{x\rightarrow-\infty}\frac{-2x+1}{3x-1}=\lim\limits_{x\rightarrow-\infty}\frac{-2+\frac{1}{x}}{3-\frac{1}{x}}=-\frac{2}{3}\)

\(b=\lim\limits_{x\rightarrow+\infty}\frac{\sqrt{9+\frac{1}{x}+\frac{1}{x^2}}-\sqrt{4+\frac{2}{x}+\frac{1}{x^2}}}{1+\frac{1}{x}}=\frac{\sqrt{9}-\sqrt{4}}{1}=1\)

\(c=\lim\limits_{x\rightarrow+\infty}\frac{\sqrt{1+\frac{2}{x}+\frac{3}{x^2}}+4+\frac{1}{x}}{\sqrt{4+\frac{1}{x^2}}+\frac{2}{x}-1}=\frac{1+4}{\sqrt{4}-1}=5\)

\(d=\lim\limits_{x\rightarrow+\infty}\frac{\frac{3}{x}-\frac{2}{x\sqrt{x}}+\sqrt{1-\frac{5}{x^3}}}{2+\frac{4}{x}-\frac{5}{x^2}}=\frac{1}{2}\)

Khách vãng lai đã xóa
Nguyễn Việt Lâm
15 tháng 3 2020 lúc 23:02

Bài 2:

\(a=\lim\limits_{x\rightarrow-\infty}\frac{2+\frac{1}{x}}{1-\frac{1}{x}}=2\)

\(b=\lim\limits_{x\rightarrow-\infty}\frac{2+\frac{3}{x^3}}{1-\frac{2}{x}+\frac{1}{x^3}}=2\)

\(c=\lim\limits_{x\rightarrow+\infty}\frac{x^2\left(3+\frac{1}{x^2}\right)x\left(5+\frac{3}{x}\right)}{x^3\left(2-\frac{1}{x^3}\right)x\left(1+\frac{4}{x}\right)}=\frac{15}{+\infty}=0\)

Khách vãng lai đã xóa
Phụng Nguyễn Thị
Xem chi tiết
Nguyễn Việt Lâm
13 tháng 5 2020 lúc 16:06

\(\lim\limits_{x\rightarrow+\infty}\frac{\sqrt{x^2+1}+x}{3x+5}=\lim\limits_{x\rightarrow+\infty}\frac{\sqrt{1+\frac{1}{x^2}}+1}{3+\frac{5}{x}}=\frac{2}{3}\)

lu nguyễn
Xem chi tiết
Nguyễn Việt Lâm
16 tháng 3 2020 lúc 0:01

\(a=\lim\limits_{x\rightarrow0}\frac{x^2}{x\left(\sqrt{1+x^2}+1\right)}=\lim\limits_{x\rightarrow0}\frac{x}{\sqrt{1+x^2}+1}=\frac{0}{2}=0\)

\(b=\lim\limits_{x\rightarrow1}\frac{\sqrt[3]{x+7}-2+2-\sqrt{5-x^2}}{x-1}=\lim\limits_{x\rightarrow1}\frac{\frac{x-1}{\sqrt[3]{\left(x+7\right)^2}+2\sqrt[3]{x+7}+4}+\frac{\left(x-1\right)\left(x+1\right)}{2+\sqrt{5-x^2}}}{x-1}\)

\(=\lim\limits_{x\rightarrow1}\left(\frac{1}{\sqrt[3]{\left(x+7\right)^2}+2\sqrt[3]{x+7}+4}+\frac{x+1}{2+\sqrt{5-x^2}}\right)=\frac{1}{12}+\frac{1}{2}=\frac{7}{12}\)

\(c=\lim\limits_{x\rightarrow0}\frac{2x}{x\left(\sqrt[3]{\left(1+x\right)^2}+\sqrt[3]{\left(1+x\right)\left(1-x\right)}+\sqrt[3]{\left(1-x\right)^2}\right)}=\lim\limits_{x\rightarrow0}\frac{2}{\sqrt[3]{\left(1+x\right)^2}+\sqrt[3]{\left(1+x\right)\left(1-x\right)}+\sqrt[3]{\left(1-x\right)^2}}=\frac{2}{3}\)

\(d=\frac{\sqrt[3]{6}}{0}=+\infty\)

Khách vãng lai đã xóa
Phụng Nguyễn Thị
Xem chi tiết
Nguyễn Việt Lâm
17 tháng 5 2020 lúc 16:18

Đáp án A, khi \(x\rightarrow1\) thì \(x-2< 0\) nên biểu thức không xác định

\(\Rightarrow\) Giới hạn đã cho ko tồn tại

lu nguyễn
Xem chi tiết
Nguyễn Việt Lâm
27 tháng 2 2020 lúc 10:01

Tất cả đều ko phải dạng vô định, bạn cứ thay số vào tính thôi:

\(a=\frac{sin\left(\frac{\pi}{4}\right)}{\frac{\pi}{2}}=\frac{\sqrt{2}}{\pi}\)

\(b=\frac{\sqrt[3]{3.4-4}-\sqrt{6-2}}{3}=\frac{0}{3}=0\)

\(c=0.sin\frac{1}{2}=0\)

Khách vãng lai đã xóa